LSAT 1 – Section 3 – Question 12

You need a full course to see this video. Enroll now and get started in less than a minute.

Target time: 1:34

This is question data from the 7Sage LSAT Scorer. You can score your LSATs, track your results, and analyze your performance with pretty charts and vital statistics - all with a Free Account ← sign up in less than 10 seconds

Question
QuickView
Type Tags Answer
Choices
Curve Question
Difficulty
Psg/Game/S
Difficulty
Explanation
PT1 S3 Q12
+LR
Flaw or descriptive weakening +Flaw
A
9%
152
B
6%
150
C
11%
155
D
66%
161
E
7%
152
131
147
164
+Medium 148.102 +SubsectionMedium
This page shows a recording of a live class. We're working hard to create our standard, concise explanation videos for the questions in this PrepTest. Thank you for your patience!

This is a flaw question, and we know this because of the question stem: which one of the following indicates an error in the reasoning leading to the prediction above?

We’re told that the national savings rates for certain countries have dropped. Since older people have fewer reasons to save than do younger people, this trend of decreasing savings rate will continue if the average age of the population continues to get older.

If older people have fewer reasons to save, does that mean that what their saving up for is worth less? Maybe older people just have distinct priorities, for example retirement, house, and car. All of these are huge investments. Younger people might be saving up for more things, but these things could be worth a lot less. It is not logical to assume that quantity is proportional to value.

Answer Choice (A) is descriptively accurate; however, it is not a flaw. The reason the argument is flawed has nothing to do with listing out many reasons younger people have and the strongest of those reasons. It’s about how these reasons compare to older people’s reasons.

Answer Choice (B) is not descriptively accurate. Nowhere does the argument assume that a negative savings rate can’t happen.

Answer Choice (C) is descriptively accurate but it isn’t the flaw. The average age rising comes into play in the conclusion where this is the sufficient condition: if the age rises, then the savings rate will drop lower. The author is not outright claiming that age is rising and does not need to do this.

Correct Answer Choice (D) addresses the argument overlooking the difference between amount and value. This is perfect.

Correct Answer (E) is descriptively accurate but it’s not the flaw. The different kind of taxes being compared does have anything to do with age increasing.

Take PrepTest

Review Results

Leave a Reply